Functions of Bounded Variation

Click For Summary
The discussion focuses on proving that if a function f has a continuous derivative on the interval [a,b], then the variation V(f,P) for any partition P is less than or equal to the integral of the absolute value of the derivative over that interval. By dividing the function into monotone segments and calculating the variation for each segment, it is shown that the total variation can be expressed as an integral of the derivative. This leads to the conclusion that the total variation from a to b is bounded by the integral of the absolute value of the derivative. The proof effectively demonstrates the relationship between bounded variation and continuous derivatives. The discussion emphasizes the mathematical rigor required in establishing these properties.
jdcasey9
Messages
25
Reaction score
0
1. Homework Statement [/b]
If f has a continuous derivative on [a,b], and if P is any partition of [a,b], show that V(f,P)\leq \intablf'(t)l dt. Hence, Vba\leq\intablf'(t)ldt.

Homework Equations


Monotone function \subset BV[a,b]
\sumf(ti+1)-f(ti) = lf(b) - f(a)l



The Attempt at a Solution


Let P = {a=t0 < t1 < ... < tn}. So if we divide our function into monotone segments we have:

V1(f,P) = \sumf(ti+1)-f(ti) = lf(a1) - f(a)l

V2(f,P) = \sumf(ti+1)-f(ti) = lf(a2) - f(a1)l

.
.
.

Vn(f,P) = \sumf(ti+1)-f(ti) = lf(b)- f(an-1)l

Then, treating this segments independently of the whole, we see that

v1(f,P)= lf(a)-f(a1)l=\intablf'(t)ldt = lf(a1) -f(a)l

etc.

Adding them all up V(f,P)= \intablf'(t)l dt, which satisfies our prompt.
 
Last edited:
Physics news on Phys.org
I can't get the cartesian product off of there, so please just ignore it.
 
Question: A clock's minute hand has length 4 and its hour hand has length 3. What is the distance between the tips at the moment when it is increasing most rapidly?(Putnam Exam Question) Answer: Making assumption that both the hands moves at constant angular velocities, the answer is ## \sqrt{7} .## But don't you think this assumption is somewhat doubtful and wrong?

Similar threads

  • · Replies 13 ·
Replies
13
Views
5K
  • · Replies 3 ·
Replies
3
Views
2K
  • · Replies 5 ·
Replies
5
Views
1K
  • · Replies 2 ·
Replies
2
Views
550
  • · Replies 18 ·
Replies
18
Views
3K
  • · Replies 8 ·
Replies
8
Views
2K
  • · Replies 4 ·
Replies
4
Views
1K
Replies
5
Views
3K
  • · Replies 2 ·
Replies
2
Views
2K
  • · Replies 4 ·
Replies
4
Views
4K